NU 250 Quiz #3- DM and Endocrine

Lakukan tugas rumah & ujian kamu dengan baik sekarang menggunakan Quizwiz!

Phenytoin (Dilantin) MOA

Reduces sodium and calcium and currents across neuronal membranes.

Pt feels tired, gaining wt, increasing cold sensitivity even though its now summer. Which endocrine diagnostic test should be completed first?

Free thyroxine (FT4)

Which assessment parameter has highest priority when caring for pt undergoing a water deprivation test?

Patient weight

Nurse interviews patient with a history of type 2 DM, chronic bronchitis, and osteoarthritis who has a fasting CBG of 154 mg/dL. Which patient medication may be responsible for the elevated blood glucose?

Prednisone

assesment of Dawn phenomenon

- between 2am and 6 am the patient glucose will rise. occurs more in patient with type 1

consideratoin of Glyburide, Glipizide, Glimepiride

- much more potent than 1st generation agents. lower doses are needed -beta blocker decrease the effect -avoid alcohol -avoid breastfeeding

Hypoglycemia Assessment

-<70 mg/dl blood glucose level -changes in LOC (lethargic, confusion and irritabliltly) -tremors -diaphoresis(profuse sweating causing cool clammy skin) -tachycardia -vision changes (dilated pupils(mydriasis), double vision (diplopia) or blurred vision) -weakness -seizures

Dawn phenomenon treatment

-Add or increase at-bedtime dose of insulin. -change time of administration -program insulin pump to adminster additional insulin to avoid this effect.

diabetes education

-Exercise -Diet -Self monitoring blood glucose -Medication -life style change -SUBQ injections -foot care -symptoms of hyper/hypo glycemia -

Nursing Considerations beta blocker

-assess BP and HR -it can mask of hypoglycemia -taper off do not stop abruptly

Phenytoin (Dilantin) adverse effects

-ataxia (staggering gate) - sedation -diplopia (double vision) -hypotension -gingival hyperplasia (excessive gum growth) - aplastic anemia - neutropenia -rash (may progress to stevens-johnson syndrome or toxic epidermal necrolysis, if rash occurs stop administration) -nystagmus (continuous and involuntary back and forth movement of the eyes) -hirsutism (excess hair growth)

Diabetes Interventions

-early diagnosis - exogenous (injection) insulin if needed -oral hypoglycemic (type 2- biguanides, sulfonylureas, meglitinides, alph-glucosidase inhibitors, thiazolidinediones, and dipeptidyl peptidase 4 (DPP-4). -glucose monitoring -routine exercise -diabetic diet -monitor for complications (hyper/hypo glycemia, leads to diabetic ketoacdiosis (DKA) or hyperglycemic hyperosmolar nonketotic syndrom (HHNS). chronic complications retinopathy, peripheral vasuclar disease, neuropathy and nephropathy.

Sitagliptin (Januvia) and saxagliptin (onglyza) considerations

-eat within 30 min. -gemfibrozil increase risk of hypoglycemia -short half life

hyperglycemia assessment

-elevated blood glucose levels (80-120) -polyphagia (excessive hunger or increase appetite) -polydipsia (dehydration) -polyuria (increase urine output) -hypotension -fatigue/drowsiness

Acarbose (Precose) and miglitol (glyset) side effects

-flatulence -dirarrhea -anemia -cramps

A patient is to receive methylprednisolone (Solu-Medrol) 100 mg. The label on the medication states: methylprednisolone 125 mg in 2 mL. How many milliliters will the nurse administer?

1.6 A concentration of 125 mg in 2 mL will result in 100 mg in 1.6 mL.

Hemoglobin A1c lab values

4-6%

blood glucose levels

70-110 mg/dL

In order to assist an older diabetic patient to engage in moderate daily exercise, which action is most important for the nurse to take? A. Determine what type of activities the patient enjoys. B. Remind the patient that exercise will improve self-esteem. C. Teach the patient about the effects of exercise on glucose level. D. Give the patient a list of activities that are moderate in intensity.

A - Because consistency with exercise is important, assessment for the types of exercise that the patient finds enjoyable is the most important action by the nurse in ensuring adherence to an exercise program. The other actions will also be implemented but are not the most important in improving compliance.

Which patient action indicates a good understanding of the nurse's teaching about the use of an insulin pump? A. The patient programs the pump for an insulin bolus after eating. B. The patient changes the location of the insertion site every week. C. The patient takes the pump off at bedtime and starts it again each morning. D. The patient plans for a diet that is less flexible when using the insulin pump.

A - In addition to the basal rate of insulin infusion, the patient will adjust the pump to administer a bolus after each meal, with the dosage depending on the oral intake. The insertion site should be changed every 2 or 3 days. There is more flexibility in diet and exercise when an insulin pump is used. The pump will deliver a basal insulin rate 24 hours a day.

A 32-year-old patient with diabetes is starting on intensive insulin therapy. Which type of insulin will the nurse discuss using for mealtime coverage? A. Lispro (Humalog) B. Glargine (Lantus) C. Detemir (Levemir) D. NPH (Humulin N)

A - Rapid- or short-acting insulin is used for mealtime coverage for patients receiving intensive insulin therapy. NPH, glargine, or detemir will be used as the basal insulin.

Which statement by the patient indicates a need for additional instruction in administering insulin? A. "I need to rotate injection sites among my arms, legs, and abdomen each day." B. "I can buy the 0.5 mL syringes because the line markings will be easier to see." C. "I should draw up the regular insulin first after injecting air into the NPH bottle." D. "I do not need to aspirate the plunger to check for blood before injecting insulin."

A - Rotating sites is no longer recommended because there is more consistent insulin absorption when the same site is used consistently. The other patient statements are accurate and indicate that no additional instruction is needed.

A 38-year-old patient who has type 1 diabetes plans to swim laps daily at 1:00 PM. The clinic nurse will plan to teach the patient to A. Check glucose level before, during, and after swimming. B. Delay eating the noon meal until after the swimming class. C. Increase the morning dose of neutral protamine Hagedorn (NPH) insulin. D. Time the morning insulin injection so that the peak occurs while swimming.

A - The change in exercise will affect blood glucose, and the patient will need to monitor glucose carefully to determine the need for changes in diet and insulin administration. Because exercise tends to decrease blood glucose, patients are advised to eat before exercising. Increasing the morning NPH or timing the insulin to peak during exercise may lead to hypoglycemia, especially with the increased exercise.

A 55-year-old female patient with type 2 diabetes has a nursing diagnosis of imbalanced nutrition: more than body requirements. Which goal is most important for this patient? A. The patient will reach a glycosylated hemoglobin level of less than 7%. B. The patient will follow a diet and exercise plan that results in weight loss. C. The patient will choose a diet that distributes calories throughout the day. D. The patient will state the reasons for eliminating simple sugars in the diet.

A - The complications of diabetes are related to elevated blood glucose, and the most important patient outcome is the reduction of glucose to near-normal levels. The other outcomes also are appropriate but are not as high in priority.

The nurse is caring for a 63-year-old with a possible pituitary tumor who is scheduled for a computed tomography (CT) scan with contrast. Which information about the patient is most important to discuss with the health care provider before the test? A. History of renal insufficiency B. Complains of chronic headache C. Recent bilateral visual field loss D, Blood glucose level of 134 mg/dL

A. History of renal insufficiency - Because contrast media may cause acute kidney injury in patients with poor renal function, the health care provider will need to prescribe therapies such as IV fluids to prevent this complication. The other findings are consistent with the patient's diagnosis of a pituitary tumor.

A 22-year-old patient is being seen in the clinic with increased secretion of the anterior pituitary hormones. The nurse would expect the laboratory results to show A. Increased urinary cortisol. B. Decreased serum thyroxine. C. Elevated serum aldosterone levels. D. Low urinary catecholamines excretion.

A. Increased urinary cortisol. Increased secretion of adrenocorticotropic hormone (ACTH) by the anterior pituitary gland will lead to an increase in serum and urinary cortisol levels. An increase, rather than a decrease, in thyroxine level would be expected with increased secretion of thyroid stimulating hormone (TSH) by the anterior pituitary. Aldosterone and catecholamine levels are not controlled by the anterior pituitary.

A 56-year-old female patient has an adrenocortical adenoma, causing hyperaldosteronism. The nurse providing care should a. monitor the blood pressure every 4 hours. b. elevate the patient's legs to relieve edema. c. monitor blood glucose level every 4 hours. d. order the patient a potassium-restricted diet.

ANS: A Hypertension caused by sodium retention is a common complication of hyperaldosteronism. Hyperaldosteronism does not cause an elevation in blood glucose. The patient will be hypokalemic and require potassium supplementation before surgery. Edema does not usually occur with hyperaldosteronism

A 27-yr-old patient admitted with diabetic ketoacidosis (DKA) has a serum glucose level of 732 mg/dL and serum potassium level of 3.1 mEq/L. Which action prescribed by the health care provider should the nurse take first? a. Place the patient on a cardiac monitor. b. Administer IV potassium supplements. c. Ask the patient about home insulin doses. d. Start an insulin infusion at 0.1 units/kg/hr.

ANS: A Hypokalemia can lead to potentially fatal dysrhythmias such as ventricular tachycardia and ventricular fibrillation, which would be detected with electrocardiogram (ECG) monitoring. Because potassium must be infused over at least 1 hour, the nurse should initiate cardiac monitoring before infusion of potassium. Insulin should not be administered without cardiac monitoring because insulin infusion will further decrease potassium levels. Discussion of home insulin and possible causes can wait until the patient is stabilized.

A patient with diabetic ketoacidosis is brought to the emergency department. Which prescribed action should the nurse implement first? a. Infuse 1 L of normal saline per hour. b. Give sodium bicarbonate 50 mEq IV push. c. Administer regular insulin 10 U by IV push. d. Start a regular insulin infusion at 0.1 units/kg/hr.

ANS: A The most urgent patient problem is the hypovolemia associated with diabetic ketoacidosis (DKA), and the priority is to infuse IV fluids. The other actions can be done after the infusion of normal saline is initiated.

Which nursing intervention is appropriate for a child with type 1 diabetes who is experiencing deficient fluid volume related to abnormal fluid losses through diuresis and emesis? Select all that apply. a. Initiate IV access. b. Begin IV fluid replacement with normal saline. c. Begin IV fluid replacement with D51/2NS. d. Weigh on arrival to the unit and then every other day. e. Maintain strict intake and output monitoring.

ANS: A, B, E Feedback Correct IV access should always be obtained on a hospitalized child with dehydration and a history of type 1 diabetes. Maintaining circulation is a priority nursing intervention. If the child is vomiting and unable to maintain adequate hydration, fluid volume replacement/rehydration is needed. Normal saline is the initial IV rehydration fluid, followed by half-normal saline. Maintaining strict intake and output is essential in calculating rehydration status. Incorrect D51/2NS is not the recommended fluid for rehydration of this patient. Weighing the patient on arrival is important, but following the initial weight, the child needs to be weighed more frequently than every other day. Comparison of admission weight and a weight every 8 hours provides an indication of hydration status.

Diabetes insipidus is a disorder of the a. Anterior pituitary b. Posterior pituitary c. Adrenal cortex d. Adrenal medulla

ANS: B Feedback A The anterior pituitary produces hormones such as growth hormone, thyroid-stimulating hormone, adrenocorticotropic hormone, gonadotropin, prolactin, and melanocyte-stimulating hormone. B The principal disorder of posterior pituitary hypofunction is diabetes insipidus. C The adrenal cortex produces aldosterone, sex hormones, and glucocorticoids. D The adrenal medulla produces catecholamines.

What is the best nursing action when a child with type 1 diabetes mellitus is sweating, trembling, and pale? a. Offer the child a glass of water. b. Give the child 5 units of regular insulin subcutaneously. c. Give the child a glass of orange juice. d. Give the child glucagon subcutaneously.

ANS: C Feedback A A glass of water is not indicated in this situation. An easily digested carbohydrate is indicated when a child exhibits symptoms of hypoglycemia. B Insulin would lower blood glucose and is contraindicated for a child with hypoglycemia. C Four ounces of orange juice is an appropriate treatment for the conscious child who is exhibiting signs of hypoglycemia. D Subcutaneous injection of glucagon is used to treat hypoglycemia when the child is unconscious.

A 29-year-old woman with systemic lupus erythematosus has been prescribed 2 weeks of high-dose prednisone therapy. Which information about the prednisone is most important for the nurse to include? a. "Weigh yourself daily to monitor for weight gain caused by increased appetite." b. "A weight-bearing exercise program will help minimize the risk for osteoporosis." c. "The prednisone dose should be decreased gradually rather than stopped suddenly." d. "Call the health care provider if you experience mood alterations with the prednisone."

ANS: C Acute adrenal insufficiency may occur if exogenous corticosteroids are suddenly stopped. Mood alterations and weight gain are possible adverse effects of corticosteroid use, but these are not life-threatening effects. Osteoporosis occurs when patients take corticosteroids for longer periods.

The nurse is caring for a patient following an adrenalectomy. The highest priority in the immediate postoperative period is to a. protect the patient's skin. b. monitor for signs of infection. c. balance fluids and electrolytes. d. prevent emotional disturbances.

ANS: C After adrenalectomy, the patient is at risk for circulatory instability caused by fluctuating hormone levels, and the focus of care is to assess and maintain fluid and electrolyte status through the use of IV fluids and corticosteroids. The other goals are also important for the patient but are not as immediately life threatening as the circulatory collapse that can occur with fluid and electrolyte disturbances

Which action by a patient indicates that the home health nurse's teaching about glargine and regular insulin has been successful? a. The patient administers the glargine 30 minutes before each meal. b. The patient's family prefills the syringes with the mix of insulins weekly. c. The patient discards the open vials of glargine and regular insulin after 4 weeks. d. The patient draws up the regular insulin and then the glargine in the same syringe.

ANS: C Insulin can be stored at room temperature for 4 weeks. Glargine should not be mixed with other insulins or prefilled and stored. Short-acting regular insulin is administered before meals, and glargine is given once daily.

Which information is most important for the nurse to communicate rapidly to the health care provider about a patient admitted with possible syndrome of inappropriate antidiuretic hormone (SIADH)? a. The patient has a recent weight gain of 9 lb. b. The patient complains of dyspnea with activity. c. The patient has a urine specific gravity of 1.025. d. The patient has a serum sodium level of 118 mEq/L.

ANS: D A serum sodium of less than 120 mEq/L increases the risk for complications such as seizures and needs rapid correction. The other data are not unusual for a patient with SIADH and do not indicate the need for rapid action

A 23-year-old patient is admitted with diabetes insipidus. Which action will be most appropriate for the registered nurse (RN) to delegate to an experienced licensed practical/vocational nurse (LPN/LVN)? a. Titrate the infusion of 5% dextrose in water. b. Teach the patient how to use desmopressin (DDAVP) nasal spray. c. Assess the patient's hydration status every 8 hours. d. Administer subcutaneous DDAVP.

ANS: D Administration of medications is included in LPN/LVN education and scope of practice. Assessments, patient teaching, and titrating fluid infusions are more complex skills and should be done by the RN.

Which finding by the nurse when assessing a patient with a large pituitary adenoma is most important to report to the health care provider? a. Changes in visual field b. Milk leaking from breasts c. Blood glucose 150 mg/dL d. Nausea and projectile vomiting

ANS: D Nausea and projectile vomiting may indicate increased intracranial pressure, which will require rapid actions for diagnosis and treatment. Changes in the visual field, elevated blood glucose, and galactorrhea are common with pituitary adenoma, but these do not require rapid action to prevent life-threatening complications

Type 1 diabetes, the most common childhood endocrine disease, presents challenges to the nurse in the areas of teaching, management, and adherence. Due to recent changes in health care delivery systems, meeting the needs of a type 1 diabetic child has become even more complicated. Unless the newly diagnosed child is in diabetic ketoacidosis, the child may not be hospitalized. Is this statement true or false?

ANS: T This statement is correct. Therefore the nurse must develop a plan of care that involves child and family education and supports them in either an inpatient or in outpatient setting.

Which hormone level would the nurse expect to be elevate in response to low serum cortisol level?

Adrenocorticotropic Hormone (ACTH)

Pramlintide (Symlin)

Amylin mimetic (hypoglycemic, subQ) Decreases postprandial glucose levels (decreased gastric emptying, inhibits glucagon, satiety) May be used w/ insulin (do not mix) Oral medication absorption is delayed (stagger one hour before or 2 hours after)

Hypothalamus secretes releasing/hormones. What is the target tissue of these releasing/inhibiting hormones

Anterior Pituitary

Phenytoin (Dilantin)

Anticonvulsant

Metformin (Glucophage)

Antidiabetic

The nurse is preparing to teach a 43-year-old man who is newly diagnosed with type 2 diabetes about home management of the disease. Which action should the nurse take first? a. Ask the patient's family to participate in the diabetes education program. b. Assess the patient's perception of what it means to have diabetes mellitus. c. Demonstrate how to check glucose using capillary blood glucose monitoring. d. Discuss the need for the patient to actively participate in diabetes management.

B - Before planning teaching, the nurse should assess the patient's interest in and ability to self-manage the diabetes. After assessing the patient, the other nursing actions may be appropriate, but planning needs to be individualized to each patient.

Which patient action indicates good understanding of the nurse's teaching about administration of aspart (NovoLog) insulin? A. The patient avoids injecting the insulin into the upper abdominal area. B. The patient cleans the skin with soap and water before insulin administration. C. The patient stores the insulin in the freezer after administering the prescribed dose. D. The patient pushes the plunger down while removing the syringe from the injection site.

B - Cleaning the skin with soap and water or with alcohol is acceptable. Insulin should not be frozen. The patient should leave the syringe in place for about 5 seconds after injection to be sure that all the insulin has been injected. The upper abdominal area is one of the preferred areas for insulin injection.

The nurse identifies a need for additional teaching when the patient who is self-monitoring blood glucose A. Washes the puncture site using warm water and soap. B. Chooses a puncture site in the center of the finger pad. C. Hangs the arm down for a minute before puncturing the site. D. Says the result of 120 mg indicates good blood sugar control.

B - The patient is taught to choose a puncture site at the side of the finger pad because there are fewer nerve endings along the side of the finger pad. The other patient actions indicate that teaching has been effective.

Which information will the nurse include when teaching a 50-year-old patient who has type 2 diabetes about glyburide (Micronase, DiaBeta, Glynase)? A. Glyburide decreases glucagon secretion from the pancreas. B. Glyburide stimulates insulin production and release from the pancreas. C. Glyburide should be taken even if the morning blood glucose level is low. D. Glyburide should not be used for 48 hours after receiving IV contrast media.

B - The sulfonylureas stimulate the production and release of insulin from the pancreas. If the glucose level is low, the patient should contact the health care provider before taking the glyburide, because hypoglycemia can occur with this class of medication. Metformin should be held for 48 hours after administration of IV contrast media, but this is not necessary for glyburide. Glucagon secretion is not affected by glyburide.

A 35-year-old female patient with a possible pituitary adenoma is scheduled for a computed tomography (CT) scan with contrast media. Which patient information is most important for the nurse to communicate to the health care provider before the test? A. Bilateral poor peripheral vision B. Allergies to iodine and shellfish C. Recent weight loss of 20 pounds D. Complaint of ongoing headaches

B. Allergies to iodine and shellfish - Because the usual contrast media is iodine-based, the health care provider will need to know about the allergy before the CT scan. The other findings are common with any mass in the brain such as a pituitary adenoma.

A 30-year-old patient seen in the emergency department for severe headache and acute confusion is found to have a serum sodium level of 118 mEq/L. The nurse will anticipate the need for which diagnostic test? A. Urinary 17-ketosteroids B. Antidiuretic hormone level C. Growth hormone stimulation test D. Adrenocorticotropic hormone level

B. Antidiuretic hormone level - Elevated levels of antidiuretic hormone will cause water retention and decrease serum sodium levels. The other tests would not be helpful in determining possible causes of the patient's hyponatremia.

During the physical examination of a 36-year-old female, the nurse finds that the patient's thyroid gland cannot be palpated. The most appropriate action by the nurse is to A. Palpate the patient's neck more deeply. B. Document that the thyroid was nonpalpable. C. Notify the health care provider immediately. D. Teach the patient about thyroid hormone testing.

B. Document that the thyroid was nonpalpable. - The thyroid is frequently nonpalpable. The nurse should simply document the finding. There is no need to notify the health care provider immediately about a normal finding. There is no indication for thyroid-stimulating hormone (TSH) testing unless there is evidence of thyroid dysfunction. Deep palpation of the neck is not appropriate.

A nurse will teach a patient who is scheduled to complete a 24-hour urine collection for 17-ketosteroids to A. Insert and maintain a retention catheter. B. Keep the specimen refrigerated or on ice. C. Drink at least 3 L of fluid during the 24 hours. D. Void and save that specimen to start the collection.

B. Keep the specimen refrigerated or on ice. - The specimen must be kept on ice or refrigerated until the collection is finished. Voided or catheterized specimens are acceptable for the test. The initial voided specimen is discarded. There is no fluid intake requirement for the 24-hour collection.

Corticosteroids can affect blood glucose results. The other information will be provided to the health care provider but will not affect the test results. A. The RN checks the blood pressure on both arms. B. The RN palpates the neck thoroughly to check thyroid size. C. The RN lowers the thermostat to decrease the temperature in the room. D. The RN orders nonmedicated eye drops to lubricate the patient's bulging eyes.

B. The RN palpates the neck thoroughly to check thyroid size. - Palpation can cause the release of thyroid hormones in a patient with an enlarged thyroid and should be avoided. The other actions by the new RN are appropriate when caring for a patient with an enlarged thyroid.

The nurse is caring for a 45-year-old male patient during a water deprivation test. Which finding is most important for the nurse to communicate to the health care provider? A. The patient complains of intense thirst. B. The patient has a 5-lb (2.3 kg) weight loss. C. The patient's urine osmolality does not increase. D. The patient feels dizzy when sitting on the edge of the bed.

B. The patient has a 5-lb (2.3 kg) weight loss. - A drop in the weight of more than 2 kg indicates severe dehydration, and the test should be discontinued. The other assessment data are not unusual with this test.

Which statement by a nurse to a patient newly diagnosed with type 2 diabetes is correct? A. Insulin is not used to control blood glucose in patients with type 2 diabetes. B. Complications of type 2 diabetes are less serious than those of type 1 diabetes. C. Changes in diet and exercise may control blood glucose levels in type 2 diabetes. D. Type 2 diabetes is usually diagnosed when the patient is admitted with a hyperglycemic coma.

C - For some patients with type 2 diabetes, changes in lifestyle are sufficient to achieve blood glucose control. Insulin is frequently used for type 2 diabetes, complications are equally severe as for type 1 diabetes, and type 2 diabetes is usually diagnosed with routine laboratory testing or after a patient develops complications such as frequent yeast infections.

When a patient who takes metformin (Glucophage) to manage type 2 diabetes develops an allergic rash from an unknown cause, the health care provider prescribes prednisone (Deltasone). The nurse will anticipate that the patient may A. Need a diet higher in calories while receiving prednisone. B. Develop acute hypoglycemia while taking the prednisone. C. Require administration of insulin while taking prednisone. D. Have rashes caused by metformin-prednisone interactions.

C - Glucose levels increase when patients are taking corticosteroids, and insulin may be required to control blood glucose. Hypoglycemia is not a side effect of prednisone. Rashes are not an adverse effect caused by taking metformin and prednisone simultaneously. The patient may have an increased appetite when taking prednisone, but will not need a diet that is higher in calories.

The nurse determines a need for additional instruction when the patient with newly diagnosed type 1 diabetes says which of the following? A. "I can have an occasional alcoholic drink if I include it in my meal plan." B. "I will need a bedtime snack because I take an evening dose of NPH insulin." C. "I can choose any foods, as long as I use enough insulin to cover the calories." D. "I will eat something at meal times to prevent hypoglycemia, even if I am not hungry."

C - Most patients with type 1 diabetes need to plan diet choices very carefully. Patients who are using intensified insulin therapy have considerable flexibility in diet choices but still should restrict dietary intake of items such as fat, protein, and alcohol. The other patient statements are correct and indicate good understanding of the diet instruction.

A 48-year-old male patient screened for diabetes at a clinic has a fasting plasma glucose level of 120 mg/dL (6.7 mmol/L). The nurse will plan to teach the patient about A. Self-monitoring of blood glucose. B. Using low doses of regular insulin. C. Lifestyle changes to lower blood glucose. D. Effects of oral hypoglycemic medications.

C - The patient's impaired fasting glucose indicates prediabetes, and the patient should be counseled about lifestyle changes to prevent the development of type 2 diabetes. The patient with prediabetes does not require insulin or oral hypoglycemics for glucose control and does not need to self-monitor blood glucose.

A 29-year-old patient in the outpatient clinic will be scheduled for blood cortisol testing. Which instruction will the nurse provide? A. "Avoid adding any salt to your foods for 24 hours before the test." B. "You will need to lie down for 30 minutes before the blood is drawn." C. "Come to the laboratory to have the blood drawn early in the morning." D. "Do not have anything to eat or drink before the blood test is obtained."

C. "Come to the laboratory to have the blood drawn early in the morning." - Cortisol levels are usually drawn in the morning, when levels are highest. The other instructions would be given to patients who were having other endocrine testing.

Which question will provide the most useful information to a nurse who is interviewing a patient about a possible thyroid disorder? A. "What methods do you use to help cope with stress?" B. "Have you experienced any blurring or double vision?" C. "Have you had a recent unplanned weight gain or loss?" D. "Do you have to get up at night to empty your bladder?"

C. "Have you had a recent unplanned weight gain or loss?" - Because thyroid function affects metabolic rate, changes in weight may indicate hyperfunction or hypofunction of the thyroid gland. Nocturia, visual difficulty, and changes in stress level are associated with other endocrine disorders.

The nurse will teach a patient to plan to minimize physical and emotional stress while the patient is undergoing A. A water deprivation test. B. Testing for serum T3 and T4 levels. C. A 24-hour urine test for free cortisol. D. A radioactive iodine (I-131) uptake test.

C. A 24-hour urine test for free cortisol. - Physical and emotional stress can affect the results of the free cortisol test. The other tests are not impacted by stress.

A hospitalized diabetic patient received 38 U of NPH insulin at 7:00 AM. At 1:00 PM, the patient has been away from the nursing unit for 2 hours, missing the lunch delivery while awaiting a chest x-ray. To prevent hypoglycemia, the best action by the nurse is to A. Save the lunch tray for the patient's later return to the unit. B. Ask that diagnostic testing area staff to start a 5% dextrose IV. C. Send a glass of milk or orange juice to the patient in the diagnostic testing area. D. Request that if testing is further delayed, the patient be returned to the unit to eat.

D - Consistency for mealtimes assists with regulation of blood glucose, so the best option is for the patient to have lunch at the usual time. Waiting to eat until after the procedure is likely to cause hypoglycemia. Administration of an IV solution is unnecessarily invasive for the patient. A glass of milk or juice will keep the patient from becoming hypoglycemic but will cause a rapid rise in blood glucose because of the rapid absorption of the simple carbohydrate in these items.

A patient with type 2 diabetes is scheduled for a follow-up visit in the clinic several months from now. Which test will the nurse schedule to evaluate the effectiveness of treatment for the patient? A. Urine dipstick for glucose B. Oral glucose tolerance test C. Fasting blood glucose level D. Glycosylated hemoglobin level

D - The glycosylated hemoglobin (A1C or HbA1C) test shows the overall control of glucose over 90 to 120 days. A fasting blood level indicates only the glucose level at one time. Urine glucose testing is not an accurate reflection of blood glucose level and does not reflect the glucose over a prolonged time. Oral glucose tolerance testing is done to diagnose diabetes, but is not used for monitoring glucose control once diabetes has been diagnosed.

The nurse has been teaching a patient with type 2 diabetes about managing blood glucose levels and taking glipizide (Glucotrol). Which patient statement indicates a need for additional teaching? A. "If I overeat at a meal, I will still take the usual dose of medication." B. "Other medications besides the Glucotrol may affect my blood sugar." C. "When I am ill, I may have to take insulin to control my blood sugar." D. "My diabetes won't cause complications because I don't need insulin."

D - The patient should understand that type 2 diabetes places the patient at risk for many complications and that good glucose control is as important when taking oral agents as when using insulin. The other statements are accurate and indicate good understanding of the use of glipizide.

A 28-year-old male patient with type 1 diabetes reports how he manages his exercise and glucose control. Which behavior indicates that the nurse should implement additional teaching? A. The patient always carries hard candies when engaging in exercise. B. The patient goes for a vigorous walk when his glucose is 200 mg/dL. C. The patient has a peanut butter sandwich before going for a bicycle ride. D. The patient increases daily exercise when ketones are present in the urine.

D - When the patient is ketotic, exercise may result in an increase in blood glucose level. Type 1 diabetic patients should be taught to avoid exercise when ketosis is present. The other statements are correct.

Which statement by a 50-year-old female patient indicates to the nurse that further assessment of thyroid function may be necessary? A. "I notice my breasts are tender lately." B. "I am so thirsty that I drink all day long." C. "I get up several times at night to urinate." D. "I feel a lump in my throat when I swallow."

D. "I feel a lump in my throat when I swallow." - Difficulty in swallowing can occur with a goiter. Nocturia is associated with diseases such as diabetes mellitus, diabetes insipidus, or chronic kidney disease. Breast tenderness would occur with excessive gonadal hormone levels. Thirst is a sign of disease such as diabetes.

An 18-year-old male patient with a small stature is scheduled for a growth hormone stimulation test. In preparation for the test, the nurse will obtain A. Ice in a basin. B. Glargine insulin. C. A cardiac monitor. D. 50% dextrose solution.

D. 50% dextrose solution. - Hypoglycemia is induced during the growth hormone stimulation test, and the nurse should be ready to administer 50% dextrose immediately. Regular insulin is used to induce hypoglycemia (glargine is never given IV). The patient does not require cardiac monitoring during the test. Although blood samples for some tests must be kept on ice, this is not true for the growth hormone stimulation test.

A 61-year-old female patient admitted with pneumonia has a total serum calcium level of 13.3 mg/dL (3.3 mmol/L). The nurse will anticipate the need to teach the patient about testing for _____ levels. A. Calcitonin B. Catecholamine C. Thyroid hormone D. Parathyroid hormone

D. Parathyroid hormone - Parathyroid hormone is the major controller of blood calcium levels. Although calcitonin secretion is a countermechanism to parathyroid hormone, it does not play a major role in calcium balance. Catecholamine and thyroid hormone levels do not affect serum calcium level.

The nurse is caring for a group of older patients in a long term care setting. Which physical assessment findings would indicate possible endocrine dyfcuntion?

Fatigue, constipation and mental impairment

Dawn phenomenon

Early morning glucose elevation produced by the release of growth hormone, which decreases peripheral uptake of glucose resulting in elevated morning glucose levels. Admin of insulin at a later time in day will coordinate insulin peak with the hormone release.

Which complication should the nurse monitor for in a patent undergoing GH stimulation test?

Hypoglycemia

Exenatide (Byetta)

Incretin mimetic

Metformin (Glucophage) MOA

Metformin decreases hepatic glucose production and to a lesser extent, enhances insulin sensitivity in skeletal muscle. Metformin used alone generally does not cause hypoglycemia.

Nurse caring for pt after thyroidectomy. Which assessment findings would indicate the presence of possible hypoparathyroidism?

Muscle spasms and hyperactive deep tendon reflaxxes

Teaching pt thats scheduled for oral glucose tolerance test. What statement indicated further teaching in required?

The results of this test will indicate my blood sugar control over last 3 months

Pt seeks care b/c loss of 25 lb. x 6 months claims no significant diet changes. What potential problem should nurse assess the pt for?

Thyroid d/o

When teaching pt about urine study for free cortisol, what is most important for nurse to tell pt?

Try to avoid stressful situations during collection period

Pt recent medical hx indicative of DI. Nurse anticipate teaching about what diagnostic test?

Water deprivation test

lispro (humalog)

a rapid acting (15-30 min) analog of regular insulin with a short duration (3-6 hr). because its rapid onset, it can be administred immediately before eating, or after eating.

Acarbose (Precose) and miglitol (glyset)

alpha-glucosidase inhibitors delays intestinal glucose absoprtion*

beta blocker contraindications

asthma and heart block patient

Clozapine (Clozaril)

atypical antipsychotic

consideration for Phenytoin (Dilantin)

dosing is highly individualized and requires frequent monitoing of therapeutic plasma drug levels. Pt should maintain a level between 10 -20 mcg/mL. to discontine drug it must be taper down. abrupt withdrawl may cause a seizure.

Glyburide, Glipizide, Glimepiride side effects

hypoglycemia

Intermediate acting insulin

isophane NPH (Humulin N)

isophane NPH (Humulin N)

onset 1-2 hr, peak 4-12 hr, duration 16-24 hr. cannot be administered at mealtime, but instead is used provide glycemic control between meals and during he night.

Clozapine (Clozaril) MOA

second generation atypical antipsychotic drug given to Pt. unresponsive to other atypical meds. for treatment of psychoiss. atypical antipsychotics are less likey to cause extrapyramidal symptoms than typical antipshchotics like haloperidol.

Glyburide, Glipizide, Glimepiride

second generation sulfonylureas

Phenytoin (Dilantin) indications

seizures

Prednisone (Deltasone)

synthetic glucocorticoids med. used to treat inflammatory disease and to prevent organ tranplant rejection.

Glyburide, Glipizide, Glimepiride indications

type 2 diabetes

Pioglitazone (Thiazolidinediones) indications

type 2 diabetes

Pramlintide (Symlin) indications

type 2 diabetes

Repaglinide and Nateglinide (Meglitinides)

type 2 diabetes

Sitagliptin (Januvia) and saxagliptin (onglyza) indications

type 2 diabetes

indicatons Acarbose (Precose) and miglitol (glyset)

type 2 diabetes

Sitagliptin (Januvia) and saxagliptin (onglyza) considerations

used as 3rd line agent

loop diuretic education

-furosemide (lasix) is the most commonly used loop diuretic. other loop diuretics include bumetanide (Bumex), torasemide (demadex), and ethacrynic acid (edecrin) -notify the provider of weakness, dizzisness, or muscle cramping -increase potassium intake -check BP daily _daily weights and monitor I and O's

Pramlintide (Symlin) considerations

-give exenatide one hour after the other meds. -adjunt therapy with other antidiabetic drugs

Loop diuretic indications

-heart failure -pulmonary edema -acute renal falilure -edema

Beta blocker indications

-hypertension -heart failure -angina pectoris

Sitagliptin (Januvia) and saxagliptin (onglyza) side effect

-hypoglycemia

Pramlintide (Symlin) side effects

-hypoglycemia -nausea and vomiting -diaarrhea -thyroid cancer -pancreatitis -renal failure

considerations Pramlintide (Symlin)

-hypoglycemia when combined with insulin -delayed absorption of oral drugs

consideration Pioglitazone (Thiazolidinediones)

-increased risk of bladder cancer -increased fracture risk in wormen -monitor liver enzymes

considerations of Metformin (Glucophage)

-limit alcohol consumption - hold 1 or 2 day if IV contrasts are to be used. and 48 hours after

Hypoglycemia interventions

-mild to moderate (simple carbohydrates (oral dextrose, fruit juices, candies) complex carbohydrates (whole grains, vegetables) -severe or unconscious (50% dextrose, IM or subQ injection of Gluagon

Hypoglycemia considerations

-monitor BGL every 10-15 min -give small meal after intervention

Dawn phenomenon considerations

-monitor glucose levels closely - limit carbohydrates before bed

Acarbose (Precose) and miglitol (glyset) considerations

-monitor liver function test. AST, ALT. every three months -oral glucose for pt that suffer from hypoglycemia

Pramlintide (Symlin) side effect

-nausea -skin reactions at injection site

Metformin (Glucophage) adverse effects

-nausea, - GI discomfort, -weightloss, - toxicity: lactic acidosis

Sitagliptin (Januvia) and saxagliptin (onglyza) side effects

-pancreatitis -upper respiratory infection and inflammation -steven jonnson syndrome

indicatons Clozapine (Clozaril)

-schizophrenia -levodopa induce psychosis

Pramlintide (Symlin) indicatons

-type 1 and 2 diabetes

Metformin (Glucophage) indications

-type 2 diabetes -polycystic ovary syndrome

Mixing Insulin

-verify order -roll NPH insulin _alcohol multiuse vials -inject air into NPH insulin(cloudy) -inject air into regular insulin (clear) -withdraw regular insulin units -withdraw NPH units -verify with 2nd nurse -discard if error NOTE: think strat with cloudy end with cloudy

considerations of Clozapine (Clozaril)

-weekly lab test (WBC , absoulute neutrophil count, c-reactive protein, and troponins.

Pioglitazone (Thiazolidinediones) side effects

-weight gain -heart failure -myalgia -hypoglycemia -upper respiratory infection

Clozapine (Clozaril) side effects

-weight gain -hypersalivaiton -mycoarditis -agranulocytosis -sedation -orthostatic hypotension -constipation

The nurse performs a physical assessment on a patient with possible endocrine dysfunction. Patients weight was 142 lb. 6 months ago compared with current weight of 125 lb. What % weight change will nurse document in patients health record?

12% weight loss

A patient receives aspart (NovoLog) insulin at 8:00 AM. Which time will it be most important for the nurse to monitor for symptoms of hypoglycemia? A. 10:00 AM B. 12:00 AM C. 2:00 PM D. 4:00 PM

A - The rapid-acting insulins peak in 1 to 3 hours. The patient is not at a high risk for hypoglycemia at the other listed times, although hypoglycemia may occur.

Which statements will the nurse include when teaching a patient who is scheduled for oral glucose tolerance testing in the outpatient clinic (select all that apply)? A. "You will need to avoid smoking before the test." B. "Exercise should be avoided until the testing is complete." C. "Several blood samples will be obtained during the testing." D. "You should follow a low-calorie diet the day before the test." E. "The test requires that you fast for at least 8 hours before testing."

A, C, E A. "You will need to avoid smoking before the test." C. "Several blood samples will be obtained during the testing." E. "The test requires that you fast for at least 8 hours before testing." - Smoking may affect the results of oral glucose tolerance tests. Blood samples are obtained at baseline and at 30, 60, and 120 minutes. Accuracy requires that the patient be fasting before the test. The patient should consume at least 1500 calories/day for 3 days before the test. The patient should be ambulatory and active for accurate test results.

In which order will the nurse take these steps to prepare NPH 20 units and regular insulin 2 units using the same syringe? (Put a comma and a space between each answer choice [A, B, C, D, E]). a. Rotate NPH vial. b. Withdraw regular insulin. c. Withdraw 20 units of NPH. d. Inject 20 units of air into NPH vial. e. Inject 2 units of air into regular insulin vial.

ANS: A, D, E, B, C When mixing regular insulin with NPH, it is important to avoid contact between the regular insulin and the additives in the NPH that slow the onset, peak, and duration of activity in the longer-acting insulin.

The rise in the incidence of both overweight and obese children is directly related to the increase in the number of children diagnosed with _______________.

ANS: type 2 diabetes Type 2 diabetes is an emerging problem in the pediatric population. At the time of diagnoses, approximately 50% of the beta cells and the pancreas of type 2 diabetic children are still producing insulin. Education regarding healthy dietary choices and exercise are essential in managing these children.

A goiter is an enlargement or hypertrophy of which gland? a. Thyroid b. Adrenal c. Anterior pituitary d. Posterior pituitary

ANS: A Feedback A A goiter is an enlargement or hypertrophy of the thyroid gland. B Goiter is not associated with this secretory organ. C Goiter is not associated with this secretory organ. D Goiter is not associated with this secretory organ.

When would a child diagnosed with type 1 diabetes mellitus most likely demonstrate a decreased need for insulin? a. During the "honeymoon" phase b. During adolescence c. During growth spurts d. During minor illnesses

ANS: A Feedback A During the "honeymoon" phase, which may last from a few weeks to a year or longer, the child is likely to need less insulin. B During adolescence, physical growth and hormonal changes contribute to an increase in insulin requirements. C Insulin requirements are typically increased during growth spurts. D Stress either from illness or from events in the environment can cause hyperglycemia. Insulin requirements are increased during periods of minor illness.

A child with GH deficiency is receiving GH therapy. What is the best time for the GH to be administered? a. At bedtime b. After meals c. Before meals d. On arising in the morning

ANS: A Feedback A Injections are best given at bedtime to more closely approximate the physiologic release of GH. B This time does not mimic the physiologic release of the hormone. C This time does not mimic the physiologic release of the hormone. D This time does not mimic the physiologic release of the hormone.

What information provided by the nurse would be helpful to a 15-year-old adolescent taking methimazole three times a day? a. Pill dispensers and alarms on her watch can remind her to take the medication as ordered. b. She can take the medication when she is nervous and feels she needs it. c. She can take two pills before school and one pill at dinner, which will be easier for her to remember. d. Her mother can be responsible for reminding her when it is time to take her medication.

ANS: A Feedback A Methimazole is an antithyroid medication that should be taken three times a day. Reminders will facilitate taking medication as ordered. B This medication needs to be taken regularly, not on an as-needed basis. C The dosage cannot be combined to reduce the frequency of administration. D Because of the adolescent's school schedule and activities, she, rather than her mother, needs to be responsible for her medication.

What is the primary concern for a 7-year-old child with type 1 diabetes mellitus who asks his mother not to tell anyone at school that he has diabetes? a. The child's safety b. The privacy of the child c. Development of a sense of industry d. Peer group acceptance

ANS: A Feedback A Safety is the primary issue. School personnel need to be aware of the signs and symptoms of hypoglycemia and hyperglycemia and the appropriate interventions. B Privacy is not a life-threatening concern. C The treatment of type 1 diabetes should not interfere with the school-age child's development of a sense of industry. D Peer group acceptance, along with body image, are issues for the early adolescent with type 1 diabetes. This is not of greater priority than the child's safety.

What is the best time for the nurse to assess the peak effectiveness of subcutaneously administered Regular insulin? a. Two hours after administration b. Four hours after administration c. Immediately after administration d. Thirty minutes after administration

ANS: A Feedback A The peak action for Regular (short-acting) insulin is 2 to 3 hours after subcutaneous administration. B The duration of Regular (short-acting) insulin is only 3 to 6 hours. Peak action occurs 2 to 3 hours after the insulin is administered. C Subcutaneously administered Regular (short-acting) insulin has an onset of action of 30 to 60 minutes after injection. The effectiveness of subcutaneously administered, short-acting insulin cannot be assessed immediately after administration. D Thirty minutes corresponds to the onset of action for Regular (short-acting) insulin.

Which finding for a patient who has hypothyroidism and hypertension indicates that the nurse should contact the health care provider before administering levothyroxine (Synthroid)? a. Increased thyroxine (T4) level b. Blood pressure 112/62 mm Hg c. Distant and difficult to hear heart sounds d. Elevated thyroid stimulating hormone level

ANS: A An increased thyroxine level indicates the levothyroxine dose needs to be decreased. The other data are consistent with hypothyroidism and the nurse should administer the levothyroxine

Which question during the assessment of a patient who has diabetes will help the nurse identify autonomic neuropathy? a. "Do you feel bloated after eating?" b. "Have you seen any skin changes?" c. "Do you need to increase your insulin dosage when you are stressed?" d. "Have you noticed any painful new ulcerations or sores on your feet?"

ANS: A Autonomic neuropathy can cause delayed gastric emptying, which results in a bloated feeling for the patient. The other questions are also appropriate to ask but would not help in identifying autonomic neuropathy.

Which finding by the nurse when assessing a patient with Hashimoto's thyroiditis and a goiter will require the most immediate action? a. New-onset changes in the patient's voice b. Apical pulse rate at rest 112 beats/minute c. Elevation in the patient's T3 and T4 levels d. Bruit audible bilaterally over the thyroid gland

ANS: A Changes in the patient's voice indicate that the goiter is compressing the laryngeal nerve and may lead to airway compression. The other findings will also be reported but are expected with Hashimoto's thyroiditis and do not require immediate action

Which nursing action can the nurse delegate to experienced unlicensed assistive personnel (UAP) who are working in the diabetic clinic? a. Measure the ankle-brachial index. b. Check for changes in skin pigmentation. c. Assess for unilateral or bilateral foot drop. d. Ask the patient about symptoms of depression.

ANS: A Checking systolic pressure at the ankle and brachial areas and calculating the ankle-brachial index is a procedure that can be done by UAP who have been trained in the procedure. The other assessments require more education and critical thinking and should be done by the registered nurse (RN).

Which finding indicates to the nurse that the current therapies are effective for a patient with acute adrenal insufficiency? a. Increasing serum sodium levels b. Decreasing blood glucose levels c. Decreasing serum chloride levels d. Increasing serum potassium levels

ANS: A Clinical manifestations of Addison's disease include hyponatremia and an increase in sodium level indicates improvement. The other values indicate that treatment has not been effective

A female patient is scheduled for an oral glucose tolerance test. Which information from the patient's health history is important for the nurse to communicate to the health care provider regarding this test? a. The patient uses oral contraceptives. b. The patient runs several days a week. c. The patient has been pregnant three times. d. The patient has a family history of diabetes.

ANS: A Oral contraceptive use may falsely elevate oral glucose tolerance test (OGTT) values. Exercise and a family history of diabetes both can affect blood glucose but will not lead to misleading information from the OGTT. History of previous pregnancies may provide informational about gestational glucose tolerance but will not lead to misleading information from the OGTT.

The nurse has administered 4 oz of orange juice to an alert patient whose blood glucose was 62 mg/dL. Fifteen minutes later, the blood glucose is 67 mg/dL. Which action should the nurse take next? a. Give the patient 4 to 6 oz more orange juice. b. Administer the PRN glucagon (Glucagon) 1 mg IM. c. Have the patient eat some peanut butter with crackers. d. Notify the health care provider about the hypoglycemia.

ANS: A The "rule of 15" indicates that administration of quickly acting carbohydrates should be done two or three times for a conscious patient whose glucose remains less than 70 mg/dL before notifying the health care provider. More complex carbohydrates and fats may be used after the glucose has stabilized. Glucagon should be used if the patient's level of consciousness decreases so that oral carbohydrates can no longer be given.

Which information will the nurse include in teaching a female patient who has peripheral arterial disease, type 2 diabetes, and sensory neuropathy of the feet and legs? a. Choose flat-soled leather shoes. b. Set heating pads on a low temperature. c. Use callus remover for corns or calluses. d. Soak feet in warm water for an hour each day.

ANS: A The patient is taught to avoid high heels and that leather shoes are preferred. The feet should be washed, but not soaked, in warm water daily. Heating pad use should be avoided. Commercial callus and corn removers should be avoided. The patient should see a specialist to treat these problems.

The nurse is caring for a patient admitted with diabetes insipidus (DI). Which information is most important to report to the health care provider? a. The patient is confused and lethargic. b. The patient reports a recent head injury. c. The patient has a urine output of 400 mL/hr. d. The patient's urine specific gravity is 1.003.

ANS: A The patient's confusion and lethargy may indicate hypernatremia and should be addressed quickly. In addition, patients with DI compensate for fluid losses by drinking copious amounts of fluids, but a patient who is lethargic will be unable to drink enough fluids and will become hypovolemic. A high urine output, low urine specific gravity, and history of a recent head injury are consistent with diabetes insipidus, but they do not require immediate nursing action to avoid life-threatening complications

A patient who was admitted with myxedema coma and diagnosed with hypothyroidism is improving and expected to be discharged in 2 days. Which teaching strategy will be best for the nurse to use? a. Provide written reminders of self-care information. b. Offer multiple options for management of therapies. c. Ensure privacy for teaching by asking visitors to leave. d. Delay teaching until patient discharge date is confirmed.

ANS: A Written instructions will be helpful to the patient because initially the hypothyroid patient may be unable to remember to take medications and other aspects of self-care. Because the treatment regimen is somewhat complex, teaching should be initiated well before discharge. Family members or friends should be included in teaching because the hypothyroid patient is likely to forget some aspects of the treatment plan. A simpler regimen will be easier to understand until the patient is euthyroid

Which prescribed medication should the nurse administer first to a 60-year-old patient admitted to the emergency department in thyroid storm? a. Propranolol (Inderal) b. Propylthiouracil (PTU) c. Methimazole (Tapazole) d. Iodine (Lugol's solution)

ANS: A b-Adrenergic blockers work rapidly to decrease the cardiovascular manifestations of thyroid storm. The other medications take days to weeks to have an impact on thyroid function

What is the most appropriate intervention for the parents of a 6-year-old child with precocious puberty? a. Advise the parents to consider birth control for their daughter. b. Explain the importance of having the child foster relationships with same-age peers. c. Assure the child's parents that there is no increased risk for sexual abuse because of her appearance. d. Counsel parents that there is no treatment currently available for this disorder.

ANS: B Feedback A Advising the parents of a 6-year-old to put their daughter on birth control is not appropriate and will not reverse the effects of precocious puberty. B Despite the child's appearance, the child needs to be treated according to her chronologic age and to interact with children in the same age-group. An expected outcome is that the child will adjust socially by exhibiting age-appropriate behaviors and social interactions. C Parents need to be aware that there is an increased risk of sexual abuse for a child with precocious puberty. D Treatment for precocious puberty is the administration of gonadotropin-releasing hormone blocker, which slows or reverses the development of secondary sexual characteristics and slows rapid growth and bone aging.

Exophthalmos (protruding eyeballs) may occur in children with which condition? a. Hypothyroidism b. Hyperthyroidism c. Hypoparathyroidism d. Hyperparathyroidism

ANS: B Feedback A Hypothyroidism is not associated with exophthalmos. B Exophthalmos is a clinical manifestation of hyperthyroidism. C Hypoparathyroidism is not associated with exophthalmos. D Hyperparathyroidism is not associated with exophthalmos.

A nurse is explaining growth hormone deficiency to parents of a child admitted to rule out this problem. Which metabolic alteration that is related to growth hormone deficiency should the nurse explain to the parent? a. Hypocalcemia b. Hypoglycemia c. Diabetes insipidus. d. Hyperglycemia

ANS: B Feedback A Symptoms of hypocalcemia are associated with hypoparathyroidism. B Growth hormone helps maintain blood sugar at normal levels. C Diabetes insipidus is a disorder of the posterior pituitary. Growth hormone is produced by the anterior pituitary. D Hyperglycemia results from an insufficiency of insulin, which is produced by the beta cells in the islets of Langerhans in the pancreas.

What should a nurse advise the parents of a child with type 1 diabetes mellitus who is not eating as a result of a minor illness? a. Give the child half his regular morning dose of insulin. b. Substitute simple carbohydrates or calorie-containing liquids for solid foods. c. Give the child plenty of unsweetened, clear liquids to prevent dehydration. d. Take the child directly to the emergency department.

ANS: B Feedback A The child should receive his regular dose of insulin even if he does not have an appetite. B A sick-day diet of simple carbohydrates or calorie-containing liquids will maintain normal serum glucose levels and decrease the risk of hypoglycemia. C If the child is not eating as usual, he needs calories to prevent hypoglycemia. D During periods of minor illness, the child with type 1 diabetes mellitus can be managed safely at home.

Which laboratory finding confirms that a child with type 1 diabetes is experiencing diabetic ketoacidosis? a. No urinary ketones b. Low arterial pH c. Elevated serum carbon dioxide d. Elevated serum phosphorus

ANS: B Feedback A Urinary ketones, often in large amounts, are present when a child is in diabetic ketoacidosis. B Severe insulin deficiency produces metabolic acidosis, which is indicated by a low arterial pH. C Serum carbon dioxide is decreased in diabetic ketoacidosis. D Serum phosphorus is decreased in diabetic ketoacidosis.

A neonate born with ambiguous genitalia is diagnosed with congenital adrenogenital hyperplasia. Therapeutic management includes administration of a. Vitamin D b. Cortisone c. Stool softeners d. Calcium carbonate

ANS: B Feedback A Vitamin D has no role in the therapy of adrenogenital hyperplasia. B The most common biochemical defect with congenital adrenal hyperplasia is partial or complete 21-hydroxylase deficiency. With complete deficiency, insufficient amounts of aldosterone and cortisol are produced so that circulatory collapse occurs without immediate replacement. C Stool softeners have no role in the therapy of adrenogenital hyperplasia. D Calcium carbonate has no role in the therapy of adrenogenital hyperplasia.

A neonate is displaying mottled skin, has a large fontanel and tongue, is lethargic, and is having difficulty feeding. The nurse recognizes that this is most suggestive of a. Hypocalcemia b. Hypothyroidism c. Hypoglycemia d. Phenylketonuria (PKU)

ANS: B Feedback A When hypocalcemia is present, neonates may display twitching, tremors, irritability, jitteriness, electrocardiographic changes, and, rarely, seizures. B An infant with hypothyroidism may exhibit skin mottling, a large fontanel, a large tongue, hypotonia, slow reflexes, a distended abdomen, prolonged jaundice, lethargy, constipation, feeding problems, and coldness to touch. C Hypoglycemia causes the neonate to exhibit jitteriness, poor feeding, lethargy, seizures, respiratory alterations including apnea, hypotonia, high-pitched cry, bradycardia, cyanosis, and temperature instability. D Infants with PKU may initially have digestive problems with vomiting, and they may have a musty or mousy odor to the urine, infantile eczema, hypertonia, and hyperactive behavior.

A 30-yr-old patient has a new diagnosis of type 2 diabetes. The nurse will discuss the need to schedule a dilated eye examination a. every 2 years. c. when the patient is 39 years old. b. as soon as possible. d. within the first year after diagnosis.

ANS: B Because many patients have some diabetic retinopathy when they are first diagnosed with type 2 diabetes, a dilated eye examination is recommended at the time of diagnosis and annually thereafter. Patients with type 1 diabetes should have dilated eye examinations starting 5 years after they are diagnosed and then annually.

After change-of-shift report, which patient should the nurse assess first? a. A 19-yr-old patient with type 1 diabetes who has a hemoglobin A1C of 12% b. A 23-yr-old patient with type 1 diabetes who has a blood glucose of 40 mg/dL c. A 40-yr-old patient who is pregnant and whose oral glucose tolerance test is 202 mg/dL d. A 50-yr-old patient who uses exenatide (Byetta) and is complaining of acute abdominal pain

ANS: B Because the brain requires glucose to function, untreated hypoglycemia can cause unconsciousness, seizures, and death. The nurse will rapidly assess and treat the patient with low blood glucose. The other patients also have symptoms that require assessments or interventions, but they are not at immediate risk for life-threatening complications.

The cardiac telemetry unit charge nurse receives status reports from other nursing units about four patients who need cardiac monitoring. Which patient should be transferred to the cardiac unit first? a. Patient with Hashimoto's thyroiditis and a heart rate of 102 b. Patient with tetany who has a new order for IV calcium chloride c. Patient with Cushing syndrome and a blood glucose of 140 mg/dL d. Patient with Addison's disease who takes hydrocortisone twice daily

ANS: B Emergency treatment of tetany requires IV administration of calcium; ECG monitoring will be required because cardiac arrest may occur if high calcium levels result from too-rapid administration. The information about the other patients indicates that they are more stable than the patient with tetany

An unresponsive patient with type 2 diabetes is brought to the emergency department and diagnosed with hyperosmolar hyperglycemic syndrome (HHS). The nurse will anticipate the need to a. give 50% dextrose. c. initiate O2 by nasal cannula. b. insert an IV catheter. d. administer glargine (Lantus) insulin.

ANS: B HHS is initially treated with large volumes of IV fluids to correct hypovolemia. Regular insulin is administered, not a long-acting insulin. There is no indication that the patient requires O2. Dextrose solutions will increase the patient's blood glucose and would be contraindicated.

After receiving change-of-shift report about the following four patients, which patient should the nurse assess first? a. A 31-year-old female with Cushing syndrome and a blood glucose level of 244 mg/dL b. A 70-year-old female taking levothyroxine (Synthroid) who has an irregular pulse of 134 c. A 53-year-old male who has Addison's disease and is due for a scheduled dose of hydrocortisone (Solu-Cortef). d. A 22-year-old male admitted with syndrome of inappropriate antidiuretic hormone (SIADH) who has a serum sodium level of 130 mEq/L

ANS: B Initiation of thyroid replacement in older adults may cause angina and cardiac dysrhythmias. The patient's high pulse rate needs rapid investigation by the nurse to assess for and intervene with any cardiac problems. The other patients also require nursing assessment and/or actions but are not at risk for life-threatening complications

Which question will the nurse in the endocrine clinic ask to help determine a patient's risk factors for goiter? a. "How much milk do you drink?" b. "What medications are you taking?" c. "Are your immunizations up to date?" d. "Have you had any recent neck injuries?"

ANS: B Medications that contain thyroid-inhibiting substances can cause goiter. Milk intake, neck injury, and immunization history are not risk factors for goiter

A patient who had radical neck surgery to remove a malignant tumor developed hypoparathyroidism. The nurse should plan to teach the patient about a. bisphosphonates to reduce bone demineralization. b. calcium supplements to normalize serum calcium levels. c. increasing fluid intake to decrease risk for nephrolithiasis. d. including whole grains in the diet to prevent constipation.

ANS: B Oral calcium supplements are used to maintain the serum calcium in normal range and prevent the complications of hypocalcemia. Whole grain foods decrease calcium absorption and will not be recommended. Bisphosphonates will lower serum calcium levels further by preventing calcium from being reabsorbed from bone. Kidney stones are not a complication of hypoparathyroidism and low calcium levels

The nurse is taking a health history from a 29-yr-old pregnant patient at the first prenatal visit. The patient reports that she has no personal history of diabetes, but her mother has diabetes. Which action will the nurse plan to take? a. Teach the patient about administering regular insulin. b. Schedule the patient for a fasting blood glucose level. c. Teach about an increased risk for fetal problems with gestational diabetes. d. Schedule an oral glucose tolerance test for the twenty-fourth week of pregnancy.

ANS: B Patients at high risk for gestational diabetes should be screened for diabetes on the initial prenatal visit. An oral glucose tolerance test may also be used to check for diabetes, but it would be done before the twenty-fourth week. Teaching plans would depend on the outcome of a fasting blood glucose test and other tests.

Which action should the nurse take after a patient treated with intramuscular glucagon for hypoglycemia regains consciousness? a. Assess the patient for symptoms of hyperglycemia. b. Give the patient a snack of peanut butter and crackers. c. Have the patient drink a glass of orange juice or nonfat milk. d. Administer a continuous infusion of 5% dextrose for 24 hours.

ANS: B Rebound hypoglycemia can occur after glucagon administration, but having a meal containing complex carbohydrates plus protein and fat will help prevent hypoglycemia. Orange juice and nonfat milk will elevate blood glucose rapidly, but the cheese and crackers will stabilize blood glucose. Administration of IV glucose might be used in patients who were unable to take in nutrition orally. The patient should be assessed for symptoms of hypoglycemia after glucagon administration.

Which intervention will the nurse include in the plan of care for a 52-year-old male patient with syndrome of inappropriate antidiuretic hormone (SIADH)? a. Monitor for peripheral edema. b. Offer patient hard candies to suck on. c. Encourage fluids to 2 to 3 liters per day. d. Keep head of bed elevated to 30 degrees.

ANS: B Sucking on hard candies decreases thirst for a patient on fluid restriction. Patients with SIADH are on fluid restrictions of 800 to 1000 mL/day. Peripheral edema is not seen with SIADH. The head of the bed is elevated no more than 10 degrees to increase left atrial filling pressure and decrease antidiuretic hormone (ADH) release

The nurse will plan to monitor a patient diagnosed with a pheochromocytoma for a. flushing. b. headache. c. bradycardia. d. hypoglycemia.

ANS: B The classic clinical manifestations of pheochromocytoma are hypertension, tachycardia, severe headache, diaphoresis, and abdominal or chest pain. Elevated blood glucose may also occur because of sympathetic nervous system stimulation. Bradycardia and flushing would not be expected

A 37-year-old patient has just arrived in the postanesthesia recovery unit (PACU) after a thyroidectomy. Which information is most important to communicate to the surgeon? a. The patient reports 7/10 incisional pain. b. The patient has increasing neck swelling. c. The patient is sleepy and difficult to arouse. d. The patient's cardiac rate is 112 beats/minute.

ANS: B The neck swelling may lead to respiratory difficulty, and rapid intervention is needed to prevent airway obstruction. The incisional pain should be treated but is not unusual after surgery. A heart rate of 112 is not unusual in a patient who has been hyperthyroid and has just arrived in the PACU from surgery. Sleepiness in the immediate postoperative period is expected.

A 38-year-old male patient is admitted to the hospital in Addisonian crisis. Which patient statement supports a nursing diagnosis of ineffective self-health management related to lack of knowledge about management of Addison's disease? a. "I frequently eat at restaurants, and my food has a lot of added salt." b. "I had the stomach flu earlier this week, so I couldn't take the hydrocortisone." c. "I always double my dose of hydrocortisone on the days that I go for a long run." d. "I take twice as much hydrocortisone in the morning dose as I do in the afternoon."

ANS: B The need for hydrocortisone replacement is increased with stressors such as illness, and the patient needs to be taught to call the health care provider because medication and IV fluids and electrolytes may need to be given. The other patient statements indicate appropriate management of the Addison's disease.

Which laboratory value reported to the nurse by the unlicensed assistive personnel (UAP) indicates an urgent need for the nurse's assessment of the patient? a. Bedtime glucose of 140 mg/dL b. Noon blood glucose of 52 mg/dL c. Fasting blood glucose of 130 mg/dL d. 2-hr postprandial glucose of 220 mg/dL

ANS: B The nurse should assess the patient with a blood glucose level of 52 mg/dL for symptoms of hypoglycemia and give the patient a carbohydrate-containing beverage such as orange juice. The other values are within an acceptable range or not immediately dangerous for a patient with diabetes.

A 63-year-old patient with primary hyperparathyroidism has a serum phosphorus level of 1.7 mg/dL (0.55 mmol/L) and calcium of 14 mg/dL (3.5 mmol/L). Which nursing action should be included in the plan of care? a. Restrict the patient to bed rest. b. Encourage 4000 mL of fluids daily. c. Institute routine seizure precautions. d. Assess for positive Chvostek's sign.

ANS: B The patient with hypercalcemia is at risk for kidney stones, which may be prevented by a high fluid intake. Seizure precautions and monitoring for Chvostek's or Trousseau's sign are appropriate for hypocalcemic patients. The patient should engage in weight-bearing exercise to decrease calcium loss from bone

An active 32-yr-old male who has type 1 diabetes is being seen in the endocrine clinic. Which finding indicates a need for the nurse to discuss a possible a change in therapy with the health care provider? a. Hemoglobin A1C level of 6.2% b. Blood pressure of 140/88 mmHg c. Heart rate at rest of 58 beats/minute d. High density lipoprotein (HDL) level of 65 mg/dL

ANS: B To decrease the incidence of macrovascular and microvascular problems in patients with diabetes, the goal blood pressure is usually 130/80 mm Hg. An A1C less than 6.5%, a low resting heart rate (consistent with regular aerobic exercise in a young adult), and an HDL level of 65 mg/dL all indicate that the patient's diabetes and risk factors for vascular disease are well controlled.

After a 22-year-old female patient with a pituitary adenoma has had a hypophysectomy, the nurse will teach about the need for a. sodium restriction to prevent fluid retention. b. insulin to maintain normal blood glucose levels. c. oral corticosteroids to replace endogenous cortisol. d. chemotherapy to prevent malignant tumor recurrence.

ANS: C Antidiuretic hormone (ADH), cortisol, and thyroid hormone replacement will be needed for life after hypophysectomy. Without the effects of adrenocorticotropic hormone (ACTH) and cortisol, the blood glucose and serum sodium will be low unless cortisol is replaced. An adenoma is a benign tumor, and chemotherapy will not be needed

A patient who has diabetes and reported burning foot pain at night receives a new prescription. Which information should the nurse teach the patient about amitriptyline ? a. Amitriptyline decreases the depression caused by your foot pain. b. Amitriptyline helps prevent transmission of pain impulses to the brain. c. Amitriptyline corrects some of the blood vessel changes that cause pain. d. Amitriptyline improves sleep and makes you less aware of nighttime pain.

ANS: B Tricyclic antidepressants (TCAs) decrease the transmission of pain impulses to the spinal cord and brain. TCAs also improve sleep quality and are used for depression, but that is not the major purpose for their use in diabetic neuropathy. The blood vessel changes that contribute to neuropathy are not affected by TCAs

To monitor for complications in a patient with type 2 diabetes, which tests will the nurse in the diabetic clinic schedule at least annually (select all that apply)? a. Chest x-ray b. Blood pressure c. Serum creatinine d. Urine for microalbuminuria e. Complete blood count (CBC) f. Monofilament testing of the foot

ANS: B, C, D, F Blood pressure, serum creatinine, urine testing for microalbuminuria, and monofilament testing of the foot are recommended at least annually to screen for possible microvascular and macrovascular complications of diabetes. Chest x-ray and CBC might be ordered if the patient with diabetes presents with symptoms of respiratory or infectious problems but are not routinely included in screening.

Which children admitted to the pediatric unit would the nurse monitor closely for development of SIADH? Select all who apply. a. A newly diagnosed preschooler with type 1 diabetes b. A school-age child returning from surgery for removal of a brain tumor c. An infant with suspected meningitis d. An adolescent with blunt abdominal trauma following a car accident e. A school-age child with head trauma

ANS: B, C, E Feedback Correct Childhood SIADH usually is caused by disorders affecting the central nervous system, such as infections (meningitis), head trauma, and brain tumors. Incorrect These conditions do not usually cause SIADH.

A child is diagnosed with hypothyroidism. The nurse should expect to assess which symptoms associated with hypothyroidism? Select all that apply. a. Weight loss b. Fatigue c. Diarrhea d. Dry, thick skin e. Cold intolerance

ANS: B, D, E Feedback Correct A child with hypothyroidism will display fatigue, dry, thick skin, and cold intolerance. Incorrect Weight loss and diarrhea are signs of hyperthyroidism.

A parent asks the nurse why self-monitoring of blood glucose is being recommended for her child with diabetes. The nurse should base the explanation on the knowledge that a. It is a less expensive method of testing. b. It is not as accurate as laboratory testing. c. Children are better able to manage the diabetes. d. The parents are better able to manage the disease.

ANS: C Feedback A Blood glucose monitoring is more expensive but provides improved management. B It is as accurate as equivalent testing done in laboratories. C Blood glucose self-management has improved diabetes management and can be used successfully by children from the time of diagnosis. Insulin dosages can be adjusted based on blood sugar results. D The ability to self-test allows the child to balance diet, exercise, and insulin. The parents are partners in the process, but the child should be taught how to manage the disease.

A common clinical manifestation of juvenile hypothyroidism is a. Insomnia b. Diarrhea c. Dry skin d. Accelerated growth

ANS: C Feedback A Children with hypothyroidism are usually sleepy. B Constipation is associated with hypothyroidism. C Thick, dry skin, mental decline, cold intolerance, and weight gain are associated with juvenile hypothyroidism. D Decelerated growth is common in juvenile hypothyroidism.

New parents ask the nurse, "Why is it necessary for our baby to have the newborn blood test?" The nurse explains that the priority outcome of mandatory newborn screening for inborn errors of metabolism is a. Appropriate community referral for affected infants b. Parental education about raising a special needs child c. Early identification of serious genetically transmitted metabolic diseases d. Early identification of electrolyte imbalances

ANS: C Feedback A Community referral is appropriate after a diagnosis is made. B With early identification and treatment, serious complications such as intellectual impairment are prevented. C Early identification of hypothyroidism is basic to the prevention of intellectual impairment in the child. D Although electrolyte imbalances could occur with some of the inborn errors of metabolism, this is not the priority outcome, nor would the newborn screen detect electrolyte imbalances.

What should the nurse include in the teaching plan for parents of a child with diabetes insipidus who is receiving DDAVP? a. Increase the dosage of DDAVP as the urine specific gravity (SG) increases. b. Give DDAVP only if urine output decreases. c. The child should have free access to water and toilet facilities at school. d. Cleanse skin before administering the transdermal patch.

ANS: C Feedback A DDAVP needs to be given as ordered by the physician. If the parents are monitoring urine SG at home, they would not increase the medication dose for increased SG; the physician may order an increased dosage for very dilute urine with decreased SG. B DDAVP needs to be given continuously as ordered by the physician. C The child's teachers should be aware of the diagnosis, and the child should have free access to water and toilet facilities at school. D DDAVP is typically given intranasally or by subcutaneous injection. For nocturnal enuresis, it may be given orally.

Which comment by a 12-year-old child with type 1 diabetes indicates deficient knowledge? a. "I rotate my insulin injection sites every time I give myself an injection." b. "I keep records of my glucose levels and insulin sites and amounts." c. "I'll be glad when I can take a pill for my diabetes like my uncle does." d. "I keep Lifesavers in my school bag in case I have a low-sugar reaction."

ANS: C Feedback A Rotating injection sites is appropriate because insulin absorption varies at different sites. B Keeping records of serum glucose and insulin sites and amounts is appropriate. C Children with type 1 diabetes will require life-long insulin therapy. D Prompt treatment of hypoglycemia reduces the possibility of a severe reaction. Keeping hard candy on hand is an appropriate action.

Which sign is the nurse most likely to assess in a child with hypoglycemia? a. Urine positive for ketones and serum glucose greater than 300 mg/dL b. Normal sensorium and serum glucose greater than 160 mg/dL c. Irritability and serum glucose less than 60 mg/dL d. Increased urination and serum glucose less than 120 mg/dL

ANS: C Feedback A Serum glucose greater than 300 mg/dL and urine positive for ketones are indicative of diabetic ketoacidosis. B Normal sensorium and serum glucose greater than 160 mg/dL are associated with hyperglycemia. C Irritability and serum glucose less than 60 mg/dL are neuroglycopenic manifestations of hypoglycemia. D Increased urination is an indicator of hyperglycemia. A serum glucose level less than 120 mg/dL is within normal limits.

What is the priority nursing goal for a 14-year-old with Graves' disease? a. Relieving constipation b. Allowing the adolescent to make decisions about whether or not to take medication c. Verbalizing the importance of adherence to the medication regimen d. Developing alternative educational goals

ANS: C Feedback A The adolescent with Graves' disease is not constipated. B Adherence to the medication schedule is important to ensure optimal health and wellness. Medications should not be skipped and dose regimens should not be tapered by the child without consultation with the child's medical provider. C To adhere to the medication schedule, children need to understand that the medication must be taken two or three times per day. D The management of Graves' disease does not interfere with school attendance and does not require alternative educational plans.

At what age is sexual development in boys and girls considered to be precocious? a. Boys, 11 years; girls, 9 years b. Boys, 12 years; girls, 10 years c. Boys, 9 years; girls, 8 years d. Boys, 10 years; girls, 9 1/2 years

ANS: C Feedback A These ages fall within the expected range of pubertal onset. B These ages fall within the expected range of pubertal onset. C Manifestations of sexual development before age 9 in boys and age 8 in girls is considered precocious and should be investigated. D These ages fall within the expected range of pubertal onset.

Which sign, when exhibited by a hospitalized child, should the nurse recognize as a characteristic of diabetes insipidus? a. Weight gain b. Increased urine specific gravity c. Increased urination d. Serum sodium level of 130 mEq/L

ANS: C Feedback A Weight gain results from retention of water when there is an excessive production of antidiuretic hormone; in diabetes insipidus there is a decreased production of antidiuretic hormone. B Concentrated urine is a sign of the syndrome of inappropriate antidiuretic hormone (SIADH), in which there is an excessive production of antidiuretic hormone. C The deficiency of antidiuretic hormone associated with diabetes insipidus causes the body to excrete large volumes of dilute urine. D A deficiency of antidiuretic hormone, as with diabetes insipidus, results in an increased serum sodium concentration (greater than 145 mEq/L).

A patient has just arrived on the unit after a thyroidectomy. Which action should the nurse take first? a. Observe the dressing for bleeding. b. Check the blood pressure and pulse. c. Assess the patient's respiratory effort. d. Support the patient's head with pillows.

ANS: C Airway obstruction is a possible complication after thyroidectomy because of swelling or bleeding at the site or tetany. The priority nursing action is to assess the airway. The other actions are also part of the standard nursing care postthyroidectomy but are not as high of a priority

After the nurse has finished teaching a patient who has a new prescription for exenatide (Byetta), which patient statement indicates that the teaching has been effective? a. "I may feel hungrier than usual when I take this medicine." b. "I will not need to worry about hypoglycemia with the Byetta." c. "I should take my daily aspirin at least an hour before the Byetta." d. "I will take the pill at the same time I eat breakfast in the morning."

ANS: C Because exenatide slows gastric emptying, oral medications should be taken at least 1 hour before the exenatide to avoid slowing absorption. Exenatide is injected and increases feelings of satiety. Hypoglycemia can occur with this medication.

A 26-yr-old female with type 1 diabetes develops a sore throat and runny nose after caring for her sick toddler. The patient calls the clinic for advice about her symptoms and a blood glucose level of 210 mg/dL despite taking her usual glargine (Lantus) and lispro (Humalog) insulin. The nurse advises the patient to a. use only the lispro insulin until the symptoms are resolved. b. limit intake of calories until the glucose is less than 120 mg/dL. c. monitor blood glucose every 4 hours and notify the clinic if it continues to rise. d. decrease intake of carbohydrates until glycosylated hemoglobin is less than 7%.

ANS: C Infection and other stressors increase blood glucose levels and the patient will need to test blood glucose frequently, treat elevations appropriately with lispro insulin, and call the health care provider if glucose levels continue to be elevated. Discontinuing the glargine will contribute to hyperglycemia and may lead to diabetic ketoacidosis (DKA). Decreasing carbohydrate or caloric intake is not appropriate because the patient will need more calories when ill. Glycosylated hemoglobin testing is not used to evaluate short-term alterations in blood glucose.

When a patient with type 2 diabetes is admitted for a cholecystectomy, which nursing action can the nurse delegate to a licensed practical/vocational nurse (LPN/LVN)? a. Communicate the blood glucose level and insulin dose to the circulating nurse in surgery. b. Discuss the reason for the use of insulin therapy during the immediate postoperative period. c. Administer the prescribed lispro (Humalog) insulin before transporting the patient to surgery. d. Plan strategies to minimize the risk for hypoglycemia or hyperglycemia during the postoperative period.

ANS: C LPN/LVN education and scope of practice includes administration of insulin. Communication about patient status with other departments, planning, and patient teaching are skills that require RN education and scope of practice.

A patient with diabetes rides a bicycle to and from work every day. Which site should the nurse teach the patient to use to administer the morning insulin? a. thigh. c. abdomen. b. buttock. d. upper arm.

ANS: C Patients should be taught not to administer insulin into a site that will be exercised because exercise will increase the rate of absorption. The thigh, buttock, and arm are all exercised by riding a bicycle.

A 37-year-old patient is being admitted with a diagnosis of Cushing syndrome. Which findings will the nurse expect during the assessment? a. Chronically low blood pressure b. Bronzed appearance of the skin c. Purplish streaks on the abdomen d. Decreased axillary and pubic hair

ANS: C Purplish-red striae on the abdomen are a common clinical manifestation of Cushing syndrome. Hypotension and bronzed-appearing skin are manifestations of Addison's disease. Decreased axillary and pubic hair occur with androgen deficiency

Which information obtained by the nurse in the endocrine clinic about a patient who has been taking prednisone (Deltasone) 40 mg daily for 3 weeks is most important to report to the health care provider? a. Patient's blood pressure is 148/94 mm Hg. b. Patient has bilateral 2+ pitting ankle edema. c. Patient stopped taking the medication 2 days ago. d. Patient has not been taking the prescribed vitamin D.

ANS: C Sudden cessation of corticosteroids after taking the medication for a week or more can lead to adrenal insufficiency, with problems such as severe hypotension and hypoglycemia. The patient will need immediate evaluation by the health care provider to prevent and/or treat adrenal insufficiency. The other information will also be reported, but does not require rapid treatment

A few weeks after an 82-yr-old patient with a new diagnosis of type 2 diabetes has been placed on metformin (Glucophage) therapy and taught about appropriate diet and exercise, the home health nurse makes a visit. Which finding should the nurse promptly discuss with the health care provider? a. Hemoglobin A1C level is 7.9%. b. Last eye examination was 18 months ago. c. Glomerular filtration rate is decreased. d. Patient has questions about the prescribed diet.

ANS: C The decrease in renal function may indicate a need to adjust the dose of metformin or change to a different medication. In older patients, the goal for A1C may be higher in order to avoid complications associated with hypoglycemia. The nurse will plan on scheduling the patient for an eye examination and addressing the questions about diet, but the area for prompt intervention is the patient's decreased renal function.

Which assessment finding of a 42-year-old patient who had a bilateral adrenalectomy requires the most rapid action by the nurse? a. The blood glucose is 176 mg/dL. b. The lungs have bibasilar crackles. c. The blood pressure (BP) is 88/50 mm Hg. d. The patient reports 5/10 incisional pain.

ANS: C The decreased BP indicates possible adrenal insufficiency. The nurse should immediately notify the health care provider so that corticosteroid medications can be administered. The nurse should also address the elevated glucose, incisional pain, and crackles with appropriate collaborative or nursing actions, but prevention and treatment of acute adrenal insufficiency is the priority after adrenalectomy.

A patient who was admitted with diabetic ketoacidosis secondary to a urinary tract infection has been weaned off an insulin drip 30 minutes ago. The patient reports feeling lightheaded and sweaty. Which action should the nurse take first? a. Infuse dextrose 50% by slow IV push. b. Administer 1 mg glucagon subcutaneously. c. Obtain a glucose reading using a finger stick. d. Have the patient drink 4 ounces of orange juice.

ANS: C The patient's clinical manifestations are consistent with hypoglycemia, and the initial action should be to check the patient's glucose with a finger stick or order a stat blood glucose. If the glucose is low, the patient should ingest a rapid-acting carbohydrate, such as orange juice. Glucagon or dextrose 50% might be given if the patient's symptoms become worse or if the patient is unconscious.

After change-of-shift report, which patient will the nurse assess first? a. A 19-yr-old patient with type 1 diabetes who was admitted with possible dawn phenomenon b. A 35-yr-old patient with type 1 diabetes whose most recent blood glucose reading was 230 mg/dL c. A 60-yr-old patient with hyperosmolar hyperglycemic syndrome who has poor skin turgor and dry oral mucosa d. A 68-yr-old patient with type 2 diabetes who has severe peripheral neuropathy and complains of burning foot pain

ANS: C The patient's diagnosis of HHS and signs of dehydration indicate that the nurse should rapidly assess for signs of shock and determine whether increased fluid infusion is needed. The other patients also need assessment and intervention but do not have life-threatening complications.

A patient develops carpopedal spasms and tingling of the lips following a parathyroidectomy. Which action should the nurse take first? a. Administer the ordered muscle relaxant. b. Give the ordered oral calcium supplement. c. Have the patient rebreathe from a paper bag. d. Start the PRN oxygen at 2 L/min per cannula.

ANS: C The patient's symptoms suggest mild hypocalcemia. The symptoms of hypocalcemia will be temporarily reduced by having the patient breathe into a paper bag, which will raise the PaCO2 and create a more acidic pH. The muscle relaxant will have no impact on the ionized calcium level. Although severe hypocalcemia can cause laryngeal stridor, there is no indication that this patient is experiencing laryngeal stridor or needs oxygen. Calcium supplements will be given to normalize calcium levels quickly, but oral supplements will take time to be absorbed

Which assessment finding for a 33-year-old female patient admitted with Graves' disease requires the most rapid intervention by the nurse? a. Bilateral exophthalmos b. Heart rate 136 beats/minute c. Temperature 103.8° F (40.4° C) d. Blood pressure 166/100 mm Hg

ANS: C The patient's temperature indicates that the patient may have thyrotoxic crisis and that interventions to lower the temperature are needed immediately. The other findings also require intervention but do not indicate potentially life-threatening complications

A patient who has type 2 diabetes is being prepared for an elective coronary angiogram. Which information would the nurse anticipate might lead to rescheduling the test? a. The patient's most recent A1C was 6.5%. b. The patient's blood glucose is 128 mg/dL. c. The patient took the prescribed metformin today. d. The patient took the prescribed captopril this morning.

ANS: C To avoid lactic acidosis, metformin should be discontinued a day or 2 before the coronary angiogram and should not be used for 48 hours after IV contrast media are administered. The other patient data will also be reported but do not indicate any need to reschedule the procedure.

An 82-year-old patient in a long-term care facility has several medications prescribed. After the patient is newly diagnosed with hypothyroidism, the nurse will need to consult with the health care provider before administering a. docusate (Colace). b. ibuprofen (Motrin). c. diazepam (Valium). d. cefoxitin (Mefoxin).

ANS: C Worsening of mental status and myxedema coma can be precipitated by the use of sedatives, especially in older adults. The nurse should discuss the use of diazepam with the health care provider before administration. The other medications may be given safely to the patient

Type 1 diabetes mellitus is suspected in an adolescent. Which clinical manifestation may be present? a. Moist skin b. Weight gain c. Fluid overload d. Blurred vision

ANS: D Feedback A Dry skin, weight loss, and dehydration are clinical manifestations of type 1 diabetes mellitus. B Dry skin, weight loss, and dehydration are clinical manifestations of type 1 diabetes mellitus. C Dry skin, weight loss, and dehydration are clinical manifestations of type 1 diabetes mellitus. D Fatigue and blurred vision are clinical manifestations of type 1 diabetes mellitus.

Which is the nurse's best response to the parents of a 10-year-old child newly diagnosed with type 1 diabetes mellitus who are concerned about the child's continued participation in soccer? a. "Consider the swim team as an alternative to soccer." b. "Encourage intellectual activity rather than participation in sports." c. "It is okay to play sports such as soccer unless the weather is too hot." d. "Give the child an extra 15 to 30 g of carbohydrate snack before soccer practice."

ANS: D Feedback A Soccer is an appropriate sport for a child with type 1 diabetes as long as the child prevents hypoglycemia by eating a snack. B Participation in sports is not contraindicated for a child with type 1 diabetes. C The child with type 1 diabetes may participate in sports activities regardless of climate. D Exercise lowers blood glucose levels. A snack with 15 to 30 g of carbohydrates before exercise will decrease the risk of hypoglycemia.

The nurse is interviewing a new patient with diabetes who takes rosiglitazone (Avandia). Which information would the nurse anticipate resulting in the health care provider discontinuing the medication? a. The patient's blood pressure is 154/92. b. The patient's blood glucose is 86 mg/dL. c. The patient reports a history of emphysema. d. The patient has chest pressure when walking.

ANS: D Rosiglitazone can cause myocardial ischemia. The nurse should immediately notify the health care provider and expect orders to discontinue the medication. A blood glucose level of 86 mg/dL indicates a positive effect from the medication. Hypertension and a history of emphysema do not contraindicate this medication.

Which finding indicates a need to contact the health care provider before the nurse administers metformin (Glucophage)? a. The patient's blood glucose level is 174 mg/dL. b. The patient is scheduled for a chest x-ray in an hour. c. The patient has gained 2 lb (0.9 kg) in the past 24 hours. d. The patient's blood urea nitrogen (BUN) level is 52 mg/dL.

ANS: D The BUN indicates possible renal failure, and metformin should not be used in patients with renal failure. The other findings are not contraindications to the use of metformin.

A 44-year-old female patient with Cushing syndrome is admitted for adrenalectomy. Which intervention by the nurse will be most helpful for a nursing diagnosis of disturbed body image related to changes in appearance? a. Reassure the patient that the physical changes are very common in patients with Cushing syndrome. b. Discuss the use of diet and exercise in controlling the weight gain associated with Cushing syndrome. c. Teach the patient that the metabolic impact of Cushing syndrome is of more importance than appearance. d. Remind the patient that most of the physical changes caused by Cushing syndrome will resolve after surgery.

ANS: D The most reassuring communication to the patient is that the physical and emotional changes caused by the Cushing syndrome will resolve after hormone levels return to normal postoperatively. Reassurance that the physical changes are expected or that there are more serious physiologic problems associated with Cushing syndrome are not therapeutic responses. The patient's physiological changes are caused by the high hormone levels, not by the patient's diet or exercise choices

Which information about a 30-year-old patient who is scheduled for an oral glucose tolerance test should be reported to the health care provider before starting the test? A. The patient reports having occasional orthostatic dizziness. B. The patient takes oral corticosteroids for rheumatoid arthritis. C. The patient has had a 10-pound weight gain in the last month. D. The patient drank several glasses of water an hour previously.

B. The patient takes oral corticosteroids for rheumatoid arthritis. - Corticosteroids can affect blood glucose results. The other information will be provided to the health care provider but will not affect the test results.

A 40-year-old male patient has been newly diagnosed with type 2 diabetes mellitus. Which information about the patient will be most useful to the nurse who is helping the patient develop strategies for successful adaptation to this disease? A. Ideal weight B. Value system C. Activity level D. Visual changes

B. Value system - When dealing with a patient with a chronic condition such as diabetes, identification of the patient's values and beliefs can assist the health care team in choosing strategies for successful lifestyle change. The other information also will be useful, but is not as important in developing an individualized plan for the necessary lifestyle changes.

The nurse is assessing a 22-year-old patient experiencing the onset of symptoms of type 1 diabetes. Which question is most appropriate for the nurse to ask? A. "Are you anorexic?" B. "Is your urine dark colored?" C. "Have you lost weight lately?" D. "Do you crave sugary drinks?"

C - Weight loss occurs because the body is no longer able to absorb glucose and starts to break down protein and fat for energy. The patient is thirsty but does not necessarily crave sugar-containing fluids. Increased appetite is a classic symptom of type 1 diabetes. With the classic symptom of polyuria, urine will be very dilute.

A 60-year-old patient is taking spironolactone (Aldactone), a drug that blocks the action of aldosterone on the kidney, for hypertension. The nurse will monitor for A. Increased serum sodium. B. Decreased urinary output. C. Elevated serum potassium. D. Evidence of fluid overload.

C. Elevated serum potassium. - Because aldosterone increases the excretion of potassium, a medication that blocks aldosterone will tend to cause hyperkalemia. Aldosterone also promotes the reabsorption of sodium and water in the renal tubules, so spironolactone will tend to cause increased urine output, a decreased or normal serum sodium level, and signs of dehydration.

The nurse reviews a patient's glycosylated hemoglobin (Hb A1C) results to evaluate A. Fasting preprandial glucose levels. B. Glucose levels 2 hours after a meal. C. Glucose control over the past 90 days. D. Hypoglycemic episodes in the past 3 months.

C. Glucose control over the past 90 days. - Glycosylated hemoglobin testing measures glucose control over the last 3 months. Glucose testing before/after a meal or random testing may reveal impaired glucose tolerance and indicate prediabetes, but it is not done on patients who already have a diagnosis of diabetes. There is no test to evaluate for hypoglycemic episodes in the past.

A 44-year-old patient is admitted with tetany. Which laboratory value should the nurse monitor? A. Total protein B. Blood glucose C. Ionized calcium D. Serum phosphate

C. Ionized calcium - Tetany is associated with hypocalcemia. The other values would not be useful for this patient.

Which additional information will the nurse need to consider when reviewing the laboratory results for a patient's total calcium level? A. The blood glucose is elevated. B. The phosphate level is normal. C. The serum albumin level is low. D. The magnesium level is normal.

C. The serum albumin level is low. - Part of the total calcium is bound to albumin so hypoalbuminemia can lead to misinterpretation of calcium levels. The other laboratory values will not affect total calcium interpretation.

Which laboratory value should the nurse review to determine whether a patient's hypothyroidism is caused by a problem with the anterior pituitary gland or with the thyroid gland? A. Thyroxine (T4) level B. Triiodothyronine (T3) level C. Thyroid-stimulating hormone (TSH) level D. Thyrotropin-releasing hormone (TRH) level

C. Thyroid-stimulating hormone (TSH) level - A low TSH level indicates that the patient's hypothyroidism is caused by decreased anterior pituitary secretion of TSH. Low T3 and T4 levels are not diagnostic of the primary cause of the hypothyroidism. TRH levels indicate the function of the hypothalamus.

Surgen unable to preserve parathyroid glands during radical thyroidectomy. Which lab value should nurse diligently monitor?

Calcium levels

The nurse determines that additional instruction is needed for a 60-year-old patient with chronic syndrome of inappropriate antidiuretic hormone (SIADH) when the patient says which of the following? a. "I need to shop for foods low in sodium and avoid adding salt to food." b. "I should weigh myself daily and report any sudden weight loss or gain." c. "I need to limit my fluid intake to no more than 1 quart of liquids a day." d. "I will eat foods high in potassium because diuretics cause potassium loss."

a. "I need to shop for foods low in sodium and avoid adding salt to food." Patients with SIADH are at risk for hyponatremia, and a sodium supplement may be prescribed. The other patient statements are correct and indicate successful teaching has occurred

A 45-year-old male patient with suspected acromegaly is seen at the clinic. To assist in making the diagnosis, which question should the nurse ask? a. "Have you had a recent head injury?" b. "Do you have to wear larger shoes now?" c. "Is there a family history of acromegaly?" d. "Are you experiencing tremors or anxiety?"

b. "Do you have to wear larger shoes now?" Acromegaly causes an enlargement of the hands and feet. Head injury and family history are not risk factors for acromegaly. Tremors and anxiety are not clinical manifestations of acromegaly

A patient who had a subtotal thyroidectomy earlier today develops laryngeal stridor and a cramp in the right hand upon returning to the surgical nursing unit. Which collaborative action will the nurse anticipate next? a. Suction the patient's airway. b. Administer IV calcium gluconate. c. Plan for emergency tracheostomy. d. Prepare for endotracheal intubation.

b. Administer IV calcium gluconate. The patient's clinical manifestations of stridor and cramping are consistent with tetany caused by hypocalcemia resulting from damage to the parathyroid glands during surgery. Endotracheal intubation or tracheostomy may be needed if the calcium does not resolve the stridor. Suctioning will not correct the stridor

Which nursing assessment of a 69-year-old patient is most important to make during initiation of thyroid replacement with levothyroxine (Synthroid)? a. Fluid balance b. Apical pulse rate c. Nutritional intake d. Orientation and alertness

b. Apical pulse rate In older patients, initiation of levothyroxine therapy can increase myocardial oxygen demand and cause angina or dysrhythmias. The medication also is expected to improve mental status and fluid balance and will increase metabolic rate and nutritional needs, but these changes will not result in potentially life-threatening complications

Which nursing action will be included in the plan of care for a 55-year-old patient with Graves' disease who has exophthalmos? a. Place cold packs on the eyes to relieve pain and swelling. b. Elevate the head of the patient's bed to reduce periorbital fluid. c. Apply alternating eye patches to protect the corneas from irritation. d. Teach the patient to blink every few seconds to lubricate the corneas.

b. Elevate the head of the patient's bed to reduce periorbital fluid. The patient should sit upright as much as possible to promote fluid drainage from the periorbital area. With exophthalmos, the patient is unable to close the eyes completely to blink. Lubrication of the eyes, rather than eye patches, will protect the eyes from developing corneal scarring. The swelling of the eye is not caused by excessive blood flow to the eye, so cold packs will not be helpful

Which information will the nurse include when teaching a 50-year-old male patient about somatropin (Genotropin)? a. The medication will be needed for 3 to 6 months. b. Inject the medication subcutaneously every day. c. Blood glucose levels may decrease when taking the medication. d. Stop taking the medication if swelling of the hands or feet occurs.

b. Inject the medication subcutaneously every day. Somatropin is injected subcutaneously on a daily basis, preferably in the evening. The patient will need to continue on somatropin for life. If swelling or other common adverse effects occur, the health care provider should be notified. Growth hormone will increase blood glucose levels

The nurse is planning postoperative care for a patient who is being admitted to the surgical unit form the recovery room after transsphenoidal resection of a pituitary tumor. Which nursing action should be included? a. Palpate extremities for edema. b. Measure urine volume every hour. c. Check hematocrit every 2 hours for 8 hours. d. Monitor continuous pulse oximetry for 24 hours.

b. Measure urine volume every hour. After pituitary surgery, the patient is at risk for diabetes insipidus caused by cerebral edema. Monitoring of urine output and urine specific gravity is essential. Hemorrhage is not a common problem. There is no need to check the hematocrit hourly. The patient is at risk for dehydration, not volume overload. The patient is not at high risk for problems with oxygenation, and continuous pulse oximetry is not needed

The nurse is assessing a 41-year-old African American male patient diagnosed with a pituitary tumor causing panhypopituitarism. Assessment findings consistent with panhypopituitarism include a. high blood pressure. b. decreased facial hair. c. elevated blood glucose. d. tachycardia and cardiac palpitations.

b. decreased facial hair. Changes in male secondary sex characteristics such as decreased facial hair, testicular atrophy, diminished spermatogenesis, loss of libido, impotence, and decreased muscle mass are associated with decreases in follicle stimulating hormone (FSH) and luteinizing hormone (LH). Fasting hypoglycemia and hypotension occur in panhypopituitarism as a result of decreases in adrenocorticotropic hormone (ACTH) and cortisol. Bradycardia is likely due to the decrease in thyroid stimulating hormone (TSH) and thyroid hormones associated with panhypopituitarism

A 56-year-old patient who is disoriented and reports a headache and muscle cramps is hospitalized with possible syndrome of inappropriate antidiuretic hormone (SIADH). The nurse would expect the initial laboratory results to include a(n) a. elevated hematocrit. b. decreased serum sodium. c. low urine specific gravity. d. increased serum chloride.

b. decreased serum sodium. When water is retained, the serum sodium level will drop below normal, causing the clinical manifestations reported by the patient. The hematocrit will decrease because of the dilution caused by water retention. Urine will be more concentrated with a higher specific gravity. The serum chloride level will usually decrease along with the sodium level

The nurse determines that demeclocycline (Declomycin) is effective for a patient with syndrome of inappropriate antidiuretic hormone (SIADH) based on finding that the patient's a. weight has increased. b. urinary output is increased. c. peripheral edema is decreased. d. urine specific gravity is increased.

b. urinary output is increased. Demeclocycline blocks the action of antidiuretic hormone (ADH) on the renal tubules and increases urine output. An increase in weight or an increase in urine specific gravity indicates that the SIADH is not corrected. Peripheral edema does not occur with SIADH. A sudden weight gain without edema is a common clinical manifestation of this disorder

beta blocker

beta-adrenergic antagonists

Which information will the nurse teach a 48-year-old patient who has been newly diagnosed with Graves' disease? a. Exercise is contraindicated to avoid increasing metabolic rate. b. Restriction of iodine intake is needed to reduce thyroid activity. c. Antithyroid medications may take several months for full effect. d. Surgery will eventually be required to remove the thyroid gland.

c. Antithyroid medications may take several months for full effect. Medications used to block the synthesis of thyroid hormones may take 2 to 3 months before the full effect is seen. Large doses of iodine are used to inhibit the synthesis of thyroid hormones. Exercise using large muscle groups is encouraged to decrease the irritability and hyperactivity associated with high levels of thyroid hormones. Radioactive iodine is the most common treatment for Graves' disease although surgery may be used

A 42-year-old female patient is scheduled for transsphenoidal hypophysectomy to treat a pituitary adenoma. During preoperative teaching, the nurse instructs the patient about the need to a. cough and deep breathe every 2 hours postoperatively. b. remain on bed rest for the first 48 hours after the surgery. c. avoid brushing teeth for at least 10 days after the surgery. d. be positioned flat with sandbags at the head postoperatively.

c. avoid brushing teeth for at least 10 days after the surgery. To avoid disruption of the suture line, the patient should avoid brushing the teeth for 10 days after surgery. It is not necessary to remain on bed rest after this surgery. Coughing is discouraged because it may cause leakage of cerebrospinal fluid (CSF) from the suture line. The head of the bed should be elevated 30 degrees to reduce pressure on the sella turcica and decrease the risk for headaches

After obtaining the info below regarding a pt with Addison's disease, which prescribed action will the nurse take first? complaints of fatigue, bronze skin, poor turgor, BP 76/40, HR 126, RR 24, O2 Sats 94%, Na 123, K+ 5.1, Glucose 62 a. Give 4 oz fruit juice orally b. recheck blood glucose lvl c. infuse 5% dextrose and 0.9% saline d. Admin O2 therapy PRN

c. infuse 5% dextrose & 0.9% saline The patient's poor skin turgor, hypotension, and hyponatremia indicate an Addisonian crisis. Immediate correction of the hypovolemia and hyponatremia is needed. The other actions may also be needed but are not the initial action for the patient.

An expected nursing diagnosis for a 30-year-old patient admitted to the hospital with symptoms of diabetes insipidus is a. excess fluid volume related to intake greater than output. b. impaired gas exchange related to fluid retention in lungs. c. sleep pattern disturbance related to frequent waking to void. d. risk for impaired skin integrity related to generalized edema.

c. sleep pattern disturbance related to frequent waking to void. Nocturia occurs as a result of the polyuria caused by diabetes insipidus. Edema, excess fluid volume, and fluid retention are not expected

A 62-year-old patient with hyperthyroidism is to be treated with radioactive iodine (RAI). The nurse instructs the patient a. about radioactive precautions to take with all body secretions. b. that symptoms of hyperthyroidism should be relieved in about a week. c. that symptoms of hypothyroidism may occur as the RAI therapy takes effect. d. to discontinue the antithyroid medications taken before the radioactive therapy.

c. that symptoms of hypothyroidism may occur as the RAI therapy takes effect. There is a high incidence of postradiation hypothyroidism after RAI, and the patient should be monitored for symptoms of hypothyroidism. RAI has a delayed response, with the maximum effect not seen for 2 to 3 months, and the patient will continue to take antithyroid medications during this time. The therapeutic dose of radioactive iodine is low enough that no radiation safety precautions are needed

what is a diabetic diet

carbohydrates are composed of different forms of sugars, which contribute to elevated blood sugars. in Pt. that are deabetic, the amout of calories ingestion at one time are controlled to decrease blood sugar spikes. daily protein intake remains the same as for the general population, however, high protein diets are not recommened for weigh loss method for thos with diabetes.


Set pelajaran terkait

LANGUAGE: Avoiding Stereotypes and Sexist Language

View Set

Social studies Midterms, Chapter 15 Focus Questions, ME 1.III, ME I.V, TnT1, History: World [12] The Making of Europe, History: World [13] The Byzantine Empire, Crisis, and Recovery in the West

View Set